Hinweis auf die DSGVO: Auf unserer Seite werden keine Dritt-Anbieter-Cookies verwendet und nur Daten erfasst, welche für das Minimum an Board-Funktionalität notwendig sind.
Bevor Sie sich registrieren oder das Board verwenden, lesen Sie bitte zusätzlich die DSGVO-Erklärung, welche in der Navigationsleiste verlinkt ist.

Kurzfassung der unserer Meinung nach wichtigsten DSGVO-Punkte:
Es kann vorkommen, dass Benutzer eigenverantwortlich Videos oder sonstige Medien in ihren Beiträgen verlinken, welche beim Aufruf der Forenseite als Teil der Seite samt zugehörigem Material mitgeladen werden. Sollten Sie dies nicht wünschen, verwenden Sie beim Benutzen des Forums einen Blocker wie z.B. uMatrix, welcher das Laden von Inhaltsblöcken von Fremd-URLs effektiv unterbinden kann.
Wir blenden keine Werbung ein und schränken die Inhalte in keinster Weise bei Benutzung von Addblockern ein. Dadurch ist die Grundfunktionalität des Forums auch bei vollständigem Blockieren von Drittanbieter-Inhalten stets gegeben.

Cookies werden unsererseits nur verwendet um das Einloggen des Benutzers für die Dauer der Forenbenutzung zu speichern. Es steht dem Benutzer frei die Option 'Angemeldet bleiben' zu verwenden, damit der Cookie dauerhaft gespeichert bleibt und beim nächsten Besuch kein erneutes Einloggen mehr notwendig ist.
EMail-Adressen werden für Kontakt bei wichtigen Mitteilungen und zur Widerherstellung des Passwortes verwendet. Die verwendeten IPs können von uns ohne externe Hilfsmittel mit keiner realen Person in Verbindung gebracht werden und werden nach spätestens 7 Tagen gelöscht. Diese IPs werden höchstens verwendet um Neuanmeldungen unerwünschter oder gesperrter Nutzer zu identfizieren und zu unterbinden. Wir behalten uns daher vor bei Verdacht, die Frist für die IP-Löschung auf maximal 14 Tage zu verlängern.
Unsere Webseite läuft auf einem virtuellen Linux-Server, welcher von einem externen Anbieter gehostet wird. Etwaige Verstöße der DSGVO-Auflagen seitens dieses deutschen Hosters können wir nicht feststellen und somit auch nicht verfolgen.
Wir halten Backups unserer Datenbanken, welche in regelmäßigen Abständen als Schutz vor Katastrophen, Hackerangriffen und sonstigen Ausfällen erstellt werden. Sollte ein Nutzer die Löschung seiner Daten wünschen, betrachten wir es als Unzumutbar die Backups auch von den Daten zu befreien, da es sich hierbei um eine mehrtägiges Unterfangen handelt - dies ist für eine Einzelperson beim Betrieb eines privaten Forums nicht zumutbar möglich ohne das Backup komplett zu löschen.
Sollten Sie etwas gegen die dauerhafte anonyme Speicherung ihrer EMail-Adresse, ihres Pseudonyms und ihrer Beiträge in einem Backup haben, sehen Sie von der Registrierung in diesem Forum ab. Für Mitglieder, welche vor dem 25.05.2018 registriert waren steht jedoch das Recht im Raum, eine Löschung der Datenbank-Backups zu beantragen.



Wenn dies Ihr erster Besuch hier ist, lesen Sie bitte zunächst die FAQs sowie die wesentlichen Regeln zur Benutzung des Forums.
Um an den Diskussionen teilnehmen zu können, müssen Sie sich zunächst registrieren.

Fragen zur Bedeutung des Ereignishorizonts

Schwarze Löcher, wohl die mysteriösesten Objekte im All: Entstehung, Geometrie, Dynamik, Quantenaspekte
Benutzeravatar
tomS
Ehrenmitglied
Ehrenmitglied
Beiträge: 10670
Registriert: 19. Nov 2007, 20:29

Fragen zur Bedeutung des Ereignishorizonts

Beitrag von tomS » 26. Feb 2008, 23:57

Mir stellen sich folgende Probleme:

Zunächst gehen wir alle davon aus, dass der Schwarzsschildradius in der Schwarzschildmetrik eine reine Koordinatensingularität ist. Man kann "bessere" Koordinaten finden und die Koordinatensingularität wegtransformieren (für die Singularität im Zentrum bei gilt dies nicht!)

Dann können wir den Lichtkegel direkt am Ereignishorizont betrachten. Man stellt fest, dass alle physikalisch erlaubten Trajektorien innerhalb des Lichtkegels auf die Singularität hin verlaufen, d.h. nicht aus dem vom Ereignishorizont eingeschlossenen Volumen entkommen können. Davon merkt man jedoch beim Überschreiten des Ereignishorizonts nichts, da dies ein globaler Effekt ist, für den es rein lokal keine Anzeichen gibt.

Die Schlussfolgerung ist, dass beim Überqueren des Ereignishorizontes keine physikalisch beobachtbaren Effekte auftreten.

Soweit so gut - nun meine Probleme: Wenn Materie ins SL fällt, heizt sie sich auf und strahlt Röntgen- bzw. hochenergetische Gammastrahlung ab. Sie tut dies auf ihrem gesamten Weg ins Zentrum, sowohl innerhalb als auch außerhalb des Ereignishorizontes.

Erstes Problem: die Temperatur und damit die abgestrahlte Frequenz steigt sicher, je näher die Materie dem SL kommt. Allerdings gilt diese Betrachtung nur für den mitbewegten Beobachter. Für den weit entfernten, ruhenden Beobachter erscheint die Strahlung immer mehr rotverschoben, je näher die Materie dem SL kommt.
Frage: welcher Effekt dominiert? sehen wir die hochenergetische Strahlung direkt vom Ereignishorizont, oder wird sie durch die Rotverschienbung unsichtbar?

Zweites Problem: Die Materie wird sicher auch noch strahlen, wenn sie den Ereignishorizont überquert (die Strahlung wird diesen jedoch nie verlassen sondern i.A. ebenfalls ins SL fallen). Die direkt beim Überschreiten des Ereignishorizontes radial "nach außen" abgegebene Strahlung wird quasi auf dem Horizont "festgefroren" sein. Der Horizont "besteht" somit aus "gefangener" Strahlung. Für einen Beobachter, der sich nur infinitesimal innerhalb des Horizontes befindet, entfernt sich die Strahlung weiterhin mit Lichtgeschwindigkeit nach außen, während er nach innen fällt. Einem Beobachter, der sich noch infinitesimal außerhalb des Horizontes befindet und ihn dann durchquert, wird diese Strahlung mit Lichtgeschwindigkeit entgegeneilen.
Frage: bedeutet dies, dass jeder Beobachter, der den Horizont überquert, dies mit Lichtgeschwindigkeit (relativ zum Horizont) tut?

Drittes Problem: Man kann natürlich nur Relativgeschwindigkeiten zu anderen Objekten messen, alles andere hat keinen Sinn. D.h. die Relativgeschwindigkeit gleich der Lichtgeschwindigkeit direkt am Horizont hat nur Sinn, wenn am Horizont ein "Objekt" existiert. Dies wäre die o.g. "gefangene Strahlung", die von der bereits im SL verschwundenen Materie herstammt. Für massereiche SLs (z.B. im Zentrum von Galaxien) hätte sich diese Strahlung jedoch über viele Millionen bis Milliarden Jahre hin "angesammelt". Demnach bestünde der Horizont aus einer Art "Lichtmauer", die der ins SL stürzende Beobachter mit Lichtgeschwindigkeit durchquert.
Frage: gibt es diese "Lichtmauer" wirklich? und wenn ja, wie wirkt sich der Aufprall von ins SL stürzender Materie darauf aus?

Ich denke, die ersten beiden Fragen kann man beantworten, wenn man die Gleichungen der ART für SLs löst und sie geeignet interpretiert. Für die dritte Frage müsste man wohl die ins SL stürzende Matreie sowie die abgegeben Strahlung simulieren, das ist sicher schwieriger ... Hat jemand eine Idee?
Gruß
Tom

Der Wert eines Dialogs hängt vor allem von der Vielfalt der konkurrierenden Meinungen ab.
Sir Karl R. Popper

Benutzeravatar
wilfried
Ehrenmitglied
Ehrenmitglied
Beiträge: 2071
Registriert: 20. Aug 2006, 10:18
Wohnort: Mitten druff auf d'r Alb
Kontaktdaten:

Beitrag von wilfried » 27. Feb 2008, 09:56

Lieber Tom

ich werde dem Werner (gravi) ein pdf file zusenden und inh bitten dieses hier einzuhängen.


Das File befindet sich bereits schon länger auf dem Server und ist in unserem Quellenverzeichnis unter SL's zu finden. Doch auch hier:
http://abenteuer-universum.de/userfiles ... oecher.pdf

Gruß
gravi

Ich hatte letztes Jahr versucht mich der Schwarzen Löcher mit Hilfe einer MAPLE Berechnung zu nähern. Eventuell sind zumindets Teile daraus für dieses Thema verwendbar.

Wir werden sehen. Warten wir, bis Werner das fiel hier eingebunden hat.

Was mich bei Deiner Argumentation ist die Tatsache, dss Du einen Beobachter den EH überqueren lassen willst. Gleichzeitig sagtest Du aber richtig:
"auf dem EH eingefroren..."

Ein Beobachter kann den EH gar ich überqueren. Warten wir bis mein file hier sit und diskutieren dann weiter.
Sicher wird sich aber unser Ray dieses Thems annehmen, der ist ja mit SL ganz eng verbunden. Er hat mit diesem Thema promoviert.

Gruß

Wilfried
Die Symmetrie ist der entscheidende Ansatz Dinge zu verstehen:
-rot E - dB / (c dt) = (4 pi k ) / c
rot B - dE/ / (c dt) = (4 pi j ) / c
div B = 4 pi rho_m
div E = 4 pi rho_e

Benutzeravatar
tomS
Ehrenmitglied
Ehrenmitglied
Beiträge: 10670
Registriert: 19. Nov 2007, 20:29

Beitrag von tomS » 27. Feb 2008, 10:29

Ich stimme dir nicht ganz zu; ein Beobachter kann den Ereignishorizont durchaus überqueren.

Es gebe zwei Beobachter B1 und B2. B1 sitzt weit weg vom SL oder umkreist es auf einer stabilen Bahn. B2 bewegt sich im freien Fall auf das SL zu. B2 wird den EH in endlicher Zeit (seiner Eigenzeit) erreichen und überqueren. Er wird auch in endlicher Zeit die Singularität erreichen. Für B1 sieht es jedoch so aus, als ob B2 bei der Annäherung an den EH immer langsamer und "dunkler" = rotverschoben wird. B2 nähert sich aus Sicht von B1 dem EH in uneldich langer Zeit asymptotosch an.
Gruß
Tom

Der Wert eines Dialogs hängt vor allem von der Vielfalt der konkurrierenden Meinungen ab.
Sir Karl R. Popper

Benutzeravatar
Ray Light
hat sich hier eingelebt
hat sich hier eingelebt
Beiträge: 339
Registriert: 25. Sep 2006, 17:13
Wohnort: München

Beitrag von Ray Light » 27. Feb 2008, 13:44

Hallo zusammen,

die Feststellung ist richtig, dass beim Überqueren des Ereignishorizonts durch einen lokalen Beobachter nichts besonders geschieht. Kritisch ist dabei der Horizontbegriff, weil er mit Vorsicht zu genießen ist. Der übliche Ereignishorizont ist nicht lokal und taugt deshalb wenig. Ich diskutiere dies hier:
http://www.wissenschaft-online.de/astro ... 5.html#hor

Die Antworten auf Deine Fragen folgen aus der Analyse der Schwarzschild- bzw- Kerr-Raumzeit. Die Feldgleichung benötigt man nicht.

Zu Frage 1:
Es gibt hier keine gegeneinander arbeitenden Effekte.
Ein Beobachter am Ereignishorizont oder dahinter wird eine Blauverschiebung beobachten, d.h. die Strahlung nimmt sowohl an Energie, als auch an Intensität zu bzw. Uhren gehen schneller.
Ein externer Beobachter wird eine (Gravitations-)Rotverschiebung bzw. eine Zeitdilatation beobachten.

Zu Frage 2:
Relativisten haben bereits in den 1970er Jahren gezeigt, dass ein Testteilchen den Horizont in radialer Richtung exakt mit c überquert. Ich habe das in einem Paper von 2004 nochmal vorgerechnet:
http://arxiv.org/abs/astro-ph/0309832
(S.8, rechte Spalte)

Zu Frage 3:
Diese Art gefangene Strahlung gibt es vermutlich bei rotierenden Löchern und heißt Photonenorbit (bei Schwarzschild fallen Horizont und Photonenorbit zusammen). Dieser Orbit ist aber instabil. Es wurde darüber spekuliert, dass die Gammaphotonen dort ein Paarplasma erzeugen könnten, dass den Jet speisen könnte, siehe
http://www.wissenschaft-online.de/astro ... ml#penrose

Astronomische Beobachtungen stützen diese Hypothese bislang nicht unbedingt.

Gruß,
Ray
Wir haben verlernt uns zu wundern.

Benutzeravatar
tomS
Ehrenmitglied
Ehrenmitglied
Beiträge: 10670
Registriert: 19. Nov 2007, 20:29

Beitrag von tomS » 27. Feb 2008, 16:07

Danke! klingt alles interessant und will erstmal verdaut werden. Trotzdem ein paar Fragen:

@1: es gibt doch konkurrierende Effekte, nämlich die Tatsache, dass Materie, die mit exakt v=c den Horizont erreicht und dort mit anderer Materie zusammenstößt, sicher extrem hochenergetische Strahlung emittiert; also dass die Strahlung umso energiereicher wird, je näher am Horizont die Kollision stattfindet. D.h. zum einen habe ich immer hochenergetischere Photonen, zum anderen eine immer stärkere Rotverschiebung.

@2: v=c bezüglich was? wenn bezüglich des Horizontes, dann bedeutet dies, ich habe dort Materie auf Lichtgeschwindigkeit beschleunigt - das verstehe ich nicht. Bezüglich der Singularität kann es ja nicht sein, denn die Materie wird ja noch weiter beschleunigt, wenn sie weiter ins SL fällt.

@3: Wenn der Photonenorbit aus gefangenen Strahlung besteht, wie kann er dann einen Jet speisen? Muss ich mir das so wie bei der Hawkingstrahlung vorstellen - nach der Paarerzeugung (virtuelles Teilchenpaar) entkommt ein Teilchen im Jet, das andere fällt ins SL?

Gruß
Gruß
Tom

Der Wert eines Dialogs hängt vor allem von der Vielfalt der konkurrierenden Meinungen ab.
Sir Karl R. Popper

Benutzeravatar
wilfried
Ehrenmitglied
Ehrenmitglied
Beiträge: 2071
Registriert: 20. Aug 2006, 10:18
Wohnort: Mitten druff auf d'r Alb
Kontaktdaten:

Beitrag von wilfried » 27. Feb 2008, 19:05

Hallo Ihr beiden

diese Diskussion ist ja mal wieder hochinteressant.

Bei TOM´s Punkt 1):
zum einen habe ich immer hochenergetischere Photonen, zum anderen eine immer stärkere Rotverschiebung
kommt mir die Frage nach der extremen Materieverdichtung in den Sinn. Ist diese eventuell eine Erklärung, dass es in unmittelbarer Umgebung des EH wegen seiner grenzwertigkeit diese Blauverschiebung gibt?

Wenn es so wäre, was macht dann eigentlich die zeitliche Dehnung wieder mit der Blauverschiebung? Meine Frage basiert auf der Situation dass ja einmal gefangen im Schwarzschild SL kein Entkommen möglich ist und bei den andern SLs insbesonderem beim Kerr Typ ist es so, dass man sich an der Singularität offensichtlich vorbeimogeln kann.

Soll heissen:

wenn sich die Blauverschiebung diesseits des EH unmittelbar vor dem EH ergibt muss deren Strahlung ja durch die enorme Raumkrümmung trotzden aussen wieder sichtbar werden, nur enorm verzerrt...rot verschoben?

Wie erkenne ich dann, wes Ursache hier vorliegt?

Gruß

Wilfried
Die Symmetrie ist der entscheidende Ansatz Dinge zu verstehen:
-rot E - dB / (c dt) = (4 pi k ) / c
rot B - dE/ / (c dt) = (4 pi j ) / c
div B = 4 pi rho_m
div E = 4 pi rho_e

Benutzeravatar
tomS
Ehrenmitglied
Ehrenmitglied
Beiträge: 10670
Registriert: 19. Nov 2007, 20:29

Beitrag von tomS » 27. Feb 2008, 20:30

Ich interpretiere die Blauverschiebung anders. Sie bezieht sich nicht auf Licht, das sich vom Horizont entfernt, sondern auf Licht, das sich von außen dem Horizont nähert. Also ein Beobachter am Horizont sieht eine sehr weit entfernte Lichtquelle blauverschoben, während ein weit entfernter Beobachter eine Lichtquelle am Horizont rotverschoben sieht. Ein Beobachter direkt am Horizont sieht auch eine Lichtquelle am Horizont weder blau- noch rotverschoben. Die Änderung der Frequenz hängt ja mit den unterschiedlichen Gravitationspotentialen zusammen: kein Potentialunterschied => selbe Wellenlänge.

@Wilfried: zu deiner Idee mit dem Vorbeimogeln an der Singularität im Kerr-SL: Licht kann sich evtl. durch die Ringsingularität hindurchmogeln, jedoch sicher nicht mehr den Bereich des Ereignishorizontes verlassen. Trotzdem kommt aus dem Zentrum des Rings wohl für einen Beobachter innerhalb des Horizontes rotverschobenes Licht.

@Ray: Ich kenne die Beschreibung der Kerr-Metrik mit der Ringsingularität, dem Ereignis- und dem Cauchy-Horizont sowie der Ergosphäre. Ich habe aber auch gelesen, dass es für die Kerr-Metrik keine Innenraumlösung gibt (d.h. dass man sie mathematisch noch nicht konstruieren konnte); was bedeutet das?

Gruß
Gruß
Tom

Der Wert eines Dialogs hängt vor allem von der Vielfalt der konkurrierenden Meinungen ab.
Sir Karl R. Popper

Benutzeravatar
gravi
Site Admin
Site Admin
Beiträge: 6069
Registriert: 26. Nov 2005, 18:55
Wohnort: Münchhausen

Beitrag von gravi » 27. Feb 2008, 20:39

@Tom:

zum Thema Überschreitung des Horizonts mit exakt c:

Der Grund liegt wohl darin, dass an dieser Stelle die Weltlinien raumartig werden, hier ist dann jede beliebige Geschwindigkeit auch für materielle Körper gestattet - soweit ich weiß...

Gruß
gravi
Unser Wissen ist ein Tropfen. Was wir nicht wissen, ist ein Ozean.
Sir Isaac Newton

Benutzeravatar
tomS
Ehrenmitglied
Ehrenmitglied
Beiträge: 10670
Registriert: 19. Nov 2007, 20:29

Beitrag von tomS » 27. Feb 2008, 21:57

Das Vertauschen von Raum und Zeit ist aber doch ein rein mathematischer Artefakt. Ein frei fallender Beobachter nimmt am Horizont nichts wahr, seine Eigenzeit vergeht wie immer. D.h. seine eigene Weltlinie bleibt für ihn zeitartig. Andere Weltlinien, die sich mit seiner eigenen schneiden (also andere Beobachter, mit denen er zusammenstößt) sind am Schnittpunkt natürlich ebenfalls zeitartig.

Ich habe den Eindruck, dass sämtliche Effekte (außer die Photosphäre) reine Scheineffekte sind und dass damit der Horizont selbst in gewisser Weise selbst unbeobachtbar und unphysikalisch ist. Der Horizont erscheint zum einen nur in mathematischen Gleichungen und zum anderen wird er definiert als Grenze einer Region, aus der heraus keine Bahnkurven möglich sind. Diese letzte Definition ist aber rein negativ durch die Nicht-Existenz von etwas gekennzeichnet.

Damit stellt sich immer noch die Frage: v=c bzgl. was? Bzgl. des Horizontes kann ich nicht verstehen, da dieser offensichtlich nicht wirklich real existiert und keinen Bezugspunkt definiert. Etwas anderes außer der Photosphäre habe ich nicht, und bzgl. der ist v=c trivial, denn sie besteht ja aus Licht. Wenn es am Horizont keinen physikalische Effekt gibt, dann weiß ich auch nicht, ob ich jetzt gerade die Photosphäre durchquere oder ob ich nur Licht sehe, das wo anders herkommt.
Gruß
Tom

Der Wert eines Dialogs hängt vor allem von der Vielfalt der konkurrierenden Meinungen ab.
Sir Karl R. Popper

Benutzeravatar
wilfried
Ehrenmitglied
Ehrenmitglied
Beiträge: 2071
Registriert: 20. Aug 2006, 10:18
Wohnort: Mitten druff auf d'r Alb
Kontaktdaten:

Beitrag von wilfried » 28. Feb 2008, 20:01

Liebe Freunde

ich schliessa an TOM an:

und beim Kerr Typ rotiert auch noch die Raumzeit mit.
Damit die Frage v=c bezogen auf was??? völlig berechtigt

Ich weiss es nicht.

Gruß

Wilfried
Die Symmetrie ist der entscheidende Ansatz Dinge zu verstehen:
-rot E - dB / (c dt) = (4 pi k ) / c
rot B - dE/ / (c dt) = (4 pi j ) / c
div B = 4 pi rho_m
div E = 4 pi rho_e

Benutzeravatar
tomS
Ehrenmitglied
Ehrenmitglied
Beiträge: 10670
Registriert: 19. Nov 2007, 20:29

Beitrag von tomS » 28. Feb 2008, 20:36

Ich hab' ne Idee zu v=c; die möchte ich ellerdings erst mal begutachten lassen, bevor ich zu rechnen anfange.

Betrachten wir mal nicht die Geschwindigkeit selbst sondern das Linienelement ds². Für Licht ist ds²=0 exakt gültig. Nun kann ich für einen frei fallenden Beobachter (der Einfachheit halber radial, also keine Winkelkomponente) die Geodätengleichung lösen und für diese Geodäte ebenfalls das Linienelement berechnen. Das tue ich in einem Bezugssystem, indem das SL, genauer dessen Zentrum, in Ruhe ist. In diesem Sytem bestimme ich nun die Radialkoordinate des Ereignishorizontes r und für dieses r dann das Linienelement ds² des frei fallenden Beobachters.

Für v=c müsste ich eigentlich auch ds²=0 finden - ist das so?

Was anderes fällt mir nicht ein, da ich ja zum einen wegen der Raumkrümmung eine lokale Definition der Geschwindigkeit brauche und zum anderen als Bezugssystem nur das des Beobachters selbst oder das des SLs zur Verfügung stehen - andere Systeme mit physikalsicher Interpretation fallen mir nicht ein.
Gruß
Tom

Der Wert eines Dialogs hängt vor allem von der Vielfalt der konkurrierenden Meinungen ab.
Sir Karl R. Popper

Benutzeravatar
gravi
Site Admin
Site Admin
Beiträge: 6069
Registriert: 26. Nov 2005, 18:55
Wohnort: Münchhausen

Beitrag von gravi » 28. Feb 2008, 20:39

Ich würde mal rein impulsiv oder empirisch meinen, v=c ist bezogen auf den Horizont bzw. das was darunter ist - nicht auf einen außenstehenden Beobachter.
Hinter dem Horizont geht die Post sowieso noch schneller als mit c ab, denn nach allem was wir heute wissen, sind dort Raum und Zeit nicht mehr existent, eben nicht mehr zeitartig.

In dieser Frage hat aber Ray das letzte Wort... :wink:

Gruß
gravi
Unser Wissen ist ein Tropfen. Was wir nicht wissen, ist ein Ozean.
Sir Isaac Newton

Benutzeravatar
Ray Light
hat sich hier eingelebt
hat sich hier eingelebt
Beiträge: 339
Registriert: 25. Sep 2006, 17:13
Wohnort: München

Beitrag von Ray Light » 28. Feb 2008, 21:27

Hallo zusammen.

Bezüglich was? Jungens, steht doch alles in o.g. Papier: bezüglich des ZAMOs. Watt iss datt denn? Dett iss der Zero Angular Momentum Observer. Okay, ist nicht unbedingt verständlicher. Anschaulich bedeutet das, dass Kollege ZAMO immer mit der Kerr-Raumzeit mit rotiert und daher keinen Drehimpuls (= zero angular momentum) aufweist. Ihr müsst Euch das so vorstellen: wenn wir uns einem Kerr-Loch von außen nähern, spüren wir immer mehr die Rotation der Raumzeit. Ein Beobachter, der mit der Raumzeit rotiert und diesen Effekt daher nicht spürt, heißt ZAMO. Er wurde in den 1970er Jahren von dem Relativisten James Bardeen eingeführt und heißt deshalb auch Bardeen observer.

Schaut Euch nochmal Gleichung (27) in meinem Paper von 2004 an. v^(r) geht am Horizont gegen 1, also gegen c (weil hier G = c = 1 wie üblich bei den Relativisten gesetzt wurde). Die Klammern sagen: Hier bezieht sich die radiale Geschwindigkeit v_r auf den ZAMO.

Further reading zum ZAMO:
http://www.wissenschaft-online.de/astro ... .html#zamo

Nochmal zu Punkt 1 und den konkurrierenden Effekten:
Es bleibt bei dem was ich bereits gesagt habe. Ein Innenbeobachter sieht die Blauverschiebung; ein Außenbeobachter sieht die Rotverschiebung. Die Gravitationsrotverschiebung "macht alles platt" und dominiert etwaige Blauverschiebungen, z.B. aufgrund des Doppler-Effekts. Mathematisch liegt das an der Lapse-Funktion (alpha) in Boyer-Lindquist-Koordinaten, die am Horizont exakt null wird und damit alle anderen Faktoren zum Verschwinden bringt.

Tom schrieb:
@3: Wenn der Photonenorbit aus gefangenen Strahlung besteht, wie kann er dann einen Jet speisen? Muss ich mir das so wie bei der Hawkingstrahlung vorstellen - nach der Paarerzeugung (virtuelles Teilchenpaar) entkommt ein Teilchen im Jet, das andere fällt ins SL?
Nein, dieser Effekt hat nichts mit Hawking-Strahlung zu tun und gilt ganz im Rahmen der klassischen ART (ohne Quanteneffekte). Hochenergetische Photonen, die von außen in das Schwarze Loch "fallen", können u.U. auf dem Photonenorbit (der noch außerhalb des Horizonts liegt) eingefangen werden. Kommen nun weitere hochenergetische Photonen von außen und treffen auf die eingefangenen im Photonenorbit, so können sich Paare von Elektronen und Positronen bilden (ein Paarplasma bzw. ein leptonisches Plasma). Voraussetzung: die Energie der Photonen muss das Äquivalent an Ruheenergie für Elektron und Positron haben.
Weil das Ganze noch außerhalb des Ereignishorizont geschieht, können einige der Elektronen/Positronen dem Loch wieder entkommen und evt. einen Materiestrom speisen, der das Loch verlässt: den Jet.
Ich glaube nicht, dass dieser Mechanismus die beachtlichen Jets von radiolauten Quasaren oder von Blazaren speisen kann, weil ich mir nicht vorstellen kann, dass sich signifikant viele Photonen auf den Photonenorbit verirren. Ich kenne auch keine Ray-Tracing-Simulationen, die das belegen würden.

Letzter Punkt: Innere Kerr-Lösung. Richtig, sie wurde bislang nicht gefunden. Andererseits kommen moderne Neutronensternmetriken dem schon recht nahe, allerdings haben diese Lösungen zwei weitere Parameter ("Haare") im Vergleich zu Kerr, nämlich zusätzlich ein Massen-Quadrupolmoment und ein magnetisches Moment.
Nun, was meint man eigentlich mit einer inneren Kerr-Lösung? Es wird klar, wenn man es mit der Schwarzschild-Lösung vergleicht. Karl Schwarzschild publizierte 1916 eine Lösung der Einsteinschen Feldgleichung der ART, die die Gravitation einer Punktmasse (= Singularität) beschreibt. Diese Metrik ist das, was wir heute externe oder äußere Schwarzschild-Lösung nennen und die ein nicht rotierendes Schwarzes Loch beschreibt.
Im gleichen Jahr, 1916, veröffentlichte Schwarzschild eine Lösung der Einstein-Gleichung, die im Prinzip die Gravitation einer nicht rotierenden Flüssigkeitskugel beschreibt. Da eine Flüssigkeit Masse und Energie hat, löst diese Metrik eine kompliziertere Einstein-Gleichung als bei der äußeren Schwarzschild-Lösung (G = 0), nämlich diejenige mit einem Energie-Impuls-Tensor einer idealen, inkompressiblen Flüssigkeit auf der rechten Seite der Einstein-Gleichung. Diese innere Schwarzschild-Lösung besitzt keine zentrale (Krümmungs-)Singularität.

Es gibt keine innere Kerr-Lösung bedeutet also, dass es nicht möglich ist, auf analoge Weise eine rotierende Flüssigkeitskugel zu konstruieren, die nur die Eigenschaften Masse und Drehimpuls hat und die die Einstein-Gleichung löst. Es gibt nur die (äußere) Kerr-Lösung, die im Innern eine Ringsingularität aufweist.

Es ist auch Aussage des Robinson-Theorems, dass der Versuch eine innere Kerr-Lösung (mit nur zwei freien Parametern) finden zu wollen, scheitern muss. Siehe dazu:
http://www.wissenschaft-online.de/astro ... .html#robi

Gruß,
Ray
Wir haben verlernt uns zu wundern.

Benutzeravatar
tomS
Ehrenmitglied
Ehrenmitglied
Beiträge: 10670
Registriert: 19. Nov 2007, 20:29

Beitrag von tomS » 29. Feb 2008, 07:40

Hallo Ray,

wir sind bei meinen Fragen immer noch nicht ganz einer Meinung.

@1: ich meine hier keine Blauverschiebung aufgrund der Geometrie sondern einfach die Erzeugung immer hochenergetischerer Strahlung aus Kollisionen. Wenn Teilchen mit v=c am Horizont kollidieren, dann haben sie (im Bezugssystem des Horizonts) unendliche Energie und somit entstehen auch Photonen unendlicher Energie.

@2: v=c bezüglich was? OK, werde dein Papier nochmal genau lesen und dann für den einfacheren Fall der Schwarzsschildlösung durchrechnen. Wenn ich die Idee richtig verstehe, brauche ich dazu einen relativ zum Horizont ruhenden Beobachter. Wegen Drehimpuls=0 ist dies einfach r=2M, d.h. ich berechne die Vierergeschwindigkeit im System des SLs für ein frei fallenden Teilchen. Dann setze den Bahnparameter gleich der Eigenzeit, um eine physikalische Zeit zu bekommen.

@3: Ich hatte etwas anderes im Sinn als den von dir beschriebenen Photonenorbit (Da du erklärt hast, dass sich dieser außerhalb des Horizontes befindet, ist mir auch klar, dass man dazu keine Paarerzeugung sowie Tunneln durch den Horizont braucht - hätte ich auch selber draukommen können, denn ein Q.M. Mechanismus zur Speisung von Jets ist widersinnig).
Also nochmal: Wenn Materie ins SL fällt, dann entsteht Strahlung, und zwar entlang des gesamten Weges ins SL. Dabei gibt es drei Fälle:
r>2M: Die Strahlung entkommt ins Unendliche, allerdings rotverschoben.
r<2M: Die Strahlung erreicht schließlich ebenfalls die Singularität.
r=2M: Die Strahlung bleibt teilweise (für die zum Horizont tangentiale lichtartige Richtung) auf dem Horizont gefangen.

Gruß
Gruß
Tom

Der Wert eines Dialogs hängt vor allem von der Vielfalt der konkurrierenden Meinungen ab.
Sir Karl R. Popper

Benutzeravatar
Ray Light
hat sich hier eingelebt
hat sich hier eingelebt
Beiträge: 339
Registriert: 25. Sep 2006, 17:13
Wohnort: München

Beitrag von Ray Light » 29. Feb 2008, 17:46

Hey Tom,

ich glaube, dass Frage und Antwort konvergieren (ein Häufungspunkt?) :wink:

zu 1:
Mir ist nicht klar, was Du mit dem "Bezugssystem des Horizonts" meinst (zumal dieser nicht lokal ist) - aber egal. Entsteht hochenergetische Strahlung (über welchen Mechanismus auch immer) in der Nähe des Horizont, so wird ein Innenbeobachter, der sich gerade auf dem Weg in die Krümmungssingularität befindet, diese blauverschoben wahrnehmen; ein Außenbeobachter wird die Strahlung rotverschoben sehen.

zu 2:
Der ZAMO ruht nicht relativ zum Horizont, sondern rotiert lokal mit der Raumzeit mit. Diese Korotationsfrequenz (so genannte frame-dragging frequency, üblicherweise symbolisiert durch klein omega) hängt u.a. vom Koordinatenradius ab (ist also am Horizontradius eine andere, als vor dem Horizont).
Ich bin gespannt, ob Du die Verhältnisse "mal eben" für den Schwarzschild-Fall durchrechnen kannst, weil auch da schon einiges zu tun ist, aber ich will Dich nicht entmutigen. Jedenfalls hast Du eine Kontrolle: setze in meinem Paper einfach a = 0.

zu 3:
Ich habe schon verstanden, worauf Du ursprünglich hinaus wolltest.
So einfach wie Du es hier in der Fallunterscheidung darstellst, sind die Verhältnisse nicht! Wenn Du wissen willst, ob ein Photon im Loch verschwindet oder entkommen kann, musst Du die Nullgeodätengleichung lösen. Genau das macht das numerische Verfahren Ray Tracing (Thema meiner Diplomarbeit). Neben der Metrik (Kerr: a und M) brauchst Du zum Lösen der Gleichung Informationen über das Photon: wo startet es in der Raumzeit, in welche Richtung und welchen Drehimpuls hat es.

Gruß,
Ray
Wir haben verlernt uns zu wundern.

Benutzeravatar
tomS
Ehrenmitglied
Ehrenmitglied
Beiträge: 10670
Registriert: 19. Nov 2007, 20:29

Beitrag von tomS » 1. Mär 2008, 00:13

Ich hab' ne schlechte Nachricht: für einen Häufungspunkt müssen wir unendlich viele einzelne Punkte betrachten - und so lange wollte ich eigentlich nicht warten.

@1: mit Bezugssystem des Horizonte meine ich folgendes:
ich nehme die Bahn eines frei fallenden Objektes, das den Horizont irgendwann erreicht (für den außenstehenden Beobachter) bzw. überquert (aus Sicht des Objektes). Diese Geodäte definiert einen Punkt auf dem Horizont. Im einfachen Fall der Schwarzschildmetrik seien die beiden Winkelkoordinaten konstant, so dass ich nur noch r und t habe. Der Punkt r=2M und t=T, zu dem das Objekt den Horizont durchquert, definiert einen neuen Koordinatenursprung.

@2: ich hab das mit dem ZAMO schon verstanden. Im Falle der Schwarzschildlösung ist es doch aber trivial, dass er eben nicht mit rotieren muss, sondern bzgl. des Horizontes bzw. des o.g. Bezugssystems in Ruhe ist.

@3: auch das habe ich glaube ich verstanden. Wiederum ist es aber doch für den Fall der Schwarzschildlösung meine triviale Fallunterscheidung, oder nicht?
Gruß
Tom

Der Wert eines Dialogs hängt vor allem von der Vielfalt der konkurrierenden Meinungen ab.
Sir Karl R. Popper

Benutzeravatar
Ray Light
hat sich hier eingelebt
hat sich hier eingelebt
Beiträge: 339
Registriert: 25. Sep 2006, 17:13
Wohnort: München

Beitrag von Ray Light » 1. Mär 2008, 18:31

Hallo Tom.

Zu 1)

Tom schrieb:
ich nehme die Bahn eines frei fallenden Objektes, das den Horizont irgendwann erreicht (für den außenstehenden Beobachter)
Für einen äußeren Beobachter erreicht ein Objekt nie den Horizont ("nach unendlich langer Zeit"). Das kann man für ein Raum-Zeit-Diagramm in Schwarzschild zeigen: hier biegen sich die Hyperflächen mit t = const um den Horizont. Die Hyperfläche des Beobachters schneidet nie den Horizont.

zu 2)

Der ZAMO rotiert aus dem Unendlichen betrachtet immer mit klein omega um das Loch. Lokal rotiert er nicht, weil er mit der Raumzeit korotiert.
Da bei Schwarzschild klein omega verschwindet (wegen a = 0), rotiert auch der ZAMO betrachtet aus dem Unendlichen nicht - klar.

zu 3)

Tom schrieb:
Dabei gibt es drei Fälle:
r größer 2M: Die Strahlung entkommt ins Unendliche, allerdings rotverschoben.
r kleiner 2M: Die Strahlung erreicht schließlich ebenfalls die Singularität.
r = 2M: Die Strahlung bleibt teilweise (für die zum Horizont tangentiale lichtartige Richtung) auf dem Horizont gefangen.
Mir scheint, dass das noch nicht klar ist, weil auch in Schwarzschild Deine Fallunterscheidung nicht stimmt. Das erkennst Du ganz einfach daran, dass es Gegenbeispiele für Deine Fälle gibt. So kann durchaus ein Photon, dass aus r größer 2M kommt ins Loch fallen und eben nicht entkommen.
Schau Dir bitte mal die Kapitel in "The Mathematical Theory of Black Holes" von S. Chandrasekhar an. Der gute Mann rechnet sich bei den Geodäten in Schwarzschild und Kerr den Wolf und zeigt, dass die Behandlung nicht so trivial ist, wie Du hier darstellst. Da spielen (wie ich oben angemerkt habe) eine ganze Menge Parameter mehr (nicht nur die Radialkoordinate) eine Rolle.

Gruß,
Ray
Wir haben verlernt uns zu wundern.

Benutzeravatar
tomS
Ehrenmitglied
Ehrenmitglied
Beiträge: 10670
Registriert: 19. Nov 2007, 20:29

Beitrag von tomS » 2. Mär 2008, 12:54

Hallo Ray,

nochmal danke für deine Benühungen. Ich hab das alles etwas verkürzt dargestellt und dir deswegen mehr Arbeit gemacht als notwendig.

zu 1)
Du hast recht; Ich dachte daran, dass sich die Bahn asymptotisch einem Punkt auf dem Horizont nähert bzw. dass man sie quasi "unendlich" verlängern kann. Aber du hast natürlich recht - physikalisch wird der Punkt nie erreicht.

zu 2) sind wir uns ja einig

zu 3)
Du hast wieder recht. Ich hatte einen einfachen Fall im Sinn, wo ich versuche, einen "radialen Lichtstrahl" zu konstruieren, so dass also die Winkelkoordinaten keine Rolle spielen. Und dabei kann ich im Fall r>2M natürlich einen Fall finden, wo der Lichtstrahl ins SL fällt, und einen, wo er entkommt.
Ich hatte also ganz spezielle Lösungen der Geodätengleichung im Sinn und hatte das so nicht explizit hingeschrieben.

Ich hab aber trotzdem nochmal eine Frage: Ich hab für die Schwarzschildlösung für den einfachen Fall eines rein radialen Falls (Teilchen der Masse m) verifiziert, dass asymptotisch (also r>2M+) tatsächlich v=c gilt. Nun wollte ich aber eigentlich wissen, wie es mit der Geschwindigkeit für r<2M weitergeht, und da hab ich meine Probleme. Wegen der Koordinatensingularität bei r=2M kann ich die Bahn nicht analytisch zu r<2M fortsetzen.
Wenn ich statt dessen mit r<2M>2M erhalte ich sie aus v=0 im Unendlichen und der Energie des Teilchens, aber das klappt für r<2M nicht).
Ich müsste also ein anderes Koordinatensystem wählen. Da hab ich aber das Problem, dass es zwar Koordinatensysteme ohne Singularität bei r=2M gibt, dass sie aber mitbewegt sind und somit immer v=0 gilt. Dann weiß ich nicht, wie ich wieder so zurücktransformieren kann, dass ich v bzgl. r=2M erhalte.

Hast du einen Tip, welches Koordinatensystem ich wählen soll. Oder wie ich die Geodäte innerhalb des Ereignishorizontes an die von außen "anstückeln" kann? Schreibt der gute Herr Chandrasekhar was darüber?

Gruß
Gruß
Tom

Der Wert eines Dialogs hängt vor allem von der Vielfalt der konkurrierenden Meinungen ab.
Sir Karl R. Popper

Benutzeravatar
Ray Light
hat sich hier eingelebt
hat sich hier eingelebt
Beiträge: 339
Registriert: 25. Sep 2006, 17:13
Wohnort: München

Beitrag von Ray Light » 3. Mär 2008, 09:10

Guten Morgen,

okay, soweit so gut, Tom. Ein geeignetes Koordinatensystem für Deine Analyse unter Punkt 3 sind die Eddington-Finkelstein-Koordinaten. Transformation und Linienelement habe ich hier notiert:

http://www.wissenschaft-online.de/astro ... e.html#efk

Das gilt nur für den Schwarzschild-Fall. Für Kerr muss man eine andere Trafo wählen. Aber ich will nicht voreilig sein.

Gruß,
Ray
Wir haben verlernt uns zu wundern.

Benutzeravatar
tomS
Ehrenmitglied
Ehrenmitglied
Beiträge: 10670
Registriert: 19. Nov 2007, 20:29

Beitrag von tomS » 12. Mär 2008, 01:29

Hallo, danke für die Tips. Ich hab mal einige Zeit investiert, aber so richtig zufrieden mit den Ergenbissen bin ich nicht.

Den Ansatz mit den Eddington-Finkelstein-Koordinaten hab ich bald ad actra gelegt, denn die sind mir irgendwie suspekt. Ich habe eine logarithmische Singularität am Horizont und kann deshalb die Bedingung "am Horizont" nicht mal wirklich vernünftig formulieren.

Hab dann die Kruskal-Koordinaten ausprobiert. Da hab ich natürlich das Problem, dass die Koordinatentrf. nur implizit gegeben ist, aber das lässt sich lösen. Eine unschöne Eigenschaft ist jedoch, dass ich es bei der Formulierung der Geodätengleichung nicht schaffe, eine Gleichung erster Ordnung hinzuschreiben.
Normalerweise minimiere ich das Wirkungsintegral über ds und drücke letzteres durch -ds² aus. Dann ziehe ich das "Zeitdifferential" dt vor die Wurzel und habe ein gewöhnliches Wirkungsfunktional (über eine Lagrangefunktion, die von r und dr/dt abhängt), das ich minimieren muss. Normalerweise habe ich einen erhaltenen "Impuls" p=dH/(dr/dt) und kann daraus die Hamiltonfunktion H = p*(dr/dt) - L = const. über Legendre-Trf. berechnen. Leider funktioniert das mit H=const. nicht mehr, da L nun von beiden Koordinaten z und w abhängt; ds² hängt nämlich explizit von dz und dw, also "Raum" und "Zeit" in Kruskalkoordinaten sowie implizit über r(z,w) wieder von z und w ab. Damit ist L = L(dr/dt, r, w) "zeitabhängig",also w-abhängig, und H=const. gilt nicht mehr. Damit muss ich mich mit den Geodätengleichung zweiter Ordnung rumschlagen, aber das ist m.E. zu aufwendig.

Was ich dann probiert hab ist eine lineare Näherung am Horizont, also für z=w eine lineare Gleichung hinschreiben, d.h. zunächst von der Taylorreihe für ds² um z=w bzw. r=2M nur die Terme bis erster Ordnung behalten. Dann passiert etwas, was ich bereits am Anfang vermutet habe, nämlich dass ich eine Integrationskonstante H bekomme. Von diesem H hängt nun dz/dw ab, z wird also eine lineare Funktion von w mit einem Parameter a(H): z(w) = const + (1 + a) * (w - const). Damit gilt am Horizont z=w für geeignetes const.

H ist jetzt die erhaltene Hamiltonfunktion, denn die Abhängigkeit von w über r(z,w) in L verschwindet in der linearen Näherung am Horizont.

In Kruskal-Koordinaten habe ich einen gewöhnlichen Lichtkegel, also z=+w für den physikalischen Bereich des Kruskaldiagramms sowie z=-w für den unphysikalischen Bereich. Diese Struktur (also Öffnunsgwinkel des Lichtkegels = 90°) gilt in Kruskal-Koordinaten für alle w und z, also auch am Horizont. Das liegt am Linienelement ds² = f(r)*(-dw² + dz²). Daraus folgt ja für Lichtstrahlen dz = dw oder dz = -dw). Nichts anderes habe ich in meiner Gleichung für z(w) reproduziert.

Ich kann nun über geeignete Wahl der Integrationskonstanten H alle Trajektorien mit beliebeigem dz/dw innerhalb des Lichtkegels erhalten! Das ist auch klar: im mitbewegten System ist der Lichtkegel nicht entartet und alle zeitartigen Trajektorien sind zulässig.

Damit habe ich aber immer noch nicht verstanden, was die Aussage "jedes Objekt überquert den Horizont r=2M mit exakt Lichtgeschwindigkeit" eigentlich bedeuten soll:
1) In den Koordinatensystemen, die für r=2M "gute Koordinaten liefern" und für die ich Trajektorien über den Horizont hinweg fortsetzen kann, finde ich keine Geschwindigkeit = c oder so etwas ähnliches sondern eine Art frei fallendes Bezugssystem.
2) In dem einzigen Koordinatensystem, wo ich eine Geschwindigkeit bzgl. des Horizontes definieren könnte, werden die Koordinaten (Schwarzsschildkoordinaten) am Horizont selbst jedoch singulär und ich kann dann doch wiederum keine Geschwindigkeit am Horizont definieren!

Ich verstehe also die Aussage "jedes Objekt überquert den Horizont r=2M mit exakt Lichtgeschwindigkeit" leider immer noch nicht.

Grüße
Gruß
Tom

Der Wert eines Dialogs hängt vor allem von der Vielfalt der konkurrierenden Meinungen ab.
Sir Karl R. Popper

Skeltek
Site Admin
Site Admin
Beiträge: 5081
Registriert: 25. Mär 2008, 23:51
Wohnort: Stuttgart, Germany
Kontaktdaten:

Beitrag von Skeltek » 2. Apr 2008, 11:46

Imho wird die Rotverschiebung des Lichtes nicht durch das Entfernen vom Schwarzschildradius verursacht sondern durch die Tatsache, dass jedes Objekt das in das SW stuezt dieses mit zunehmender Geschwindigkeit tut. Ganz normaler Dopplereffekt will ich mal behaupten.

Die Meinung, dass von aussen kommendes Licht blauverschoben ist, ist eine utopische Vorstellung, die von dem Gedanken ausgeht, dass ein Beobachter direkt am Ereignisshorizont seine Position halten kann.
Licht, das von aussen zum am SR ruhenden Beobachter fliegt, wird im Grunde genommen langwelliger bei gleichbleibender Frequenz.

Sollte der Beobachter am SR nicht in der Lage sein seine Position zu halten und ohne Gegenbeschleunigung in das Loch stuerzen, wird das auf ihn zukommende Licht niederfrequenter bei gleichbleibender Wellenlaenge.

Das sind jetzt Spekulationen meinerseits, aber ich dachte hier eben beim lesen ich nutz mal die Gelegenheit und frag was der Rest hier von meinem Gedankengang haelt.

Gruesse, Skel

Benutzeravatar
Maclane
Ehrenmitglied
Ehrenmitglied
Beiträge: 1182
Registriert: 16. Jan 2006, 15:46
Wohnort: mal hier, mal dort

Beitrag von Maclane » 2. Apr 2008, 14:29

Skeltek hat geschrieben: Licht, das von aussen zum am SR ruhenden Beobachter fliegt, wird im Grunde genommen langwelliger bei gleichbleibender Frequenz.

Sollte der Beobachter am SR nicht in der Lage sein seine Position zu halten und ohne Gegenbeschleunigung in das Loch stuerzen, wird das auf ihn zukommende Licht niederfrequenter bei gleichbleibender Wellenlaenge.
Hallo und willkommen! :)

Aber du sag mal... "langwelliger bei gleicher Frequenz" geht doch gar nicht. Dann müsste das Licht ja schneller werden. Oder wie oder was? :?: :oops:

Gruß Mac
Das Gehirn ist nur so schlau wie sein Besitzer.

Skeltek
Site Admin
Site Admin
Beiträge: 5081
Registriert: 25. Mär 2008, 23:51
Wohnort: Stuttgart, Germany
Kontaktdaten:

Beitrag von Skeltek » 2. Apr 2008, 15:10

Skeltek hat geschrieben: Das sind jetzt Spekulationen meinerseits, aber ich dachte hier eben beim lesen ich nutz mal die Gelegenheit und frag was der Rest hier von meinem Gedankengang haelt.
Gruesse, Skel
Danke fuer den Willkommensgruss Maclane.
Mein Gedankengang war wie gesagt reine Spekulation meinerseits, ich hatte mir einfach gedacht, dass die Frequenz gleich bleiben muesste.
Ich wusste nicht, ob ich von einzelnen Photonen ausgehen soll oder das eintreffende Licht als Welle. Wenn ich jedoch von einer Welle ausgehe, kann diese ja nicht ihre Frequenz veraendern?
Anderseits wollt ich damit einfach vielleicht nur ein Gegenargument provozieren ^^
Erfahren wuerd ich aber trotzdem gern, ob eine Lichtwelle beim Flug zum schwarzen Loch aufgrund der gravitativen Kraefte auch eine Veraenderung seiner Ausdehung in radialer Richtung zum SW Mittelpunkt erfaehrt.

Benutzeravatar
tomS
Ehrenmitglied
Ehrenmitglied
Beiträge: 10670
Registriert: 19. Nov 2007, 20:29

Beitrag von tomS » 3. Apr 2008, 00:18

Jetzt mal langsam. Also zunächst gibt es natürlich den Dopplereffekt aufgrund von relativ zueinander bewegten Körpern (Lichtquelle und Empfänger). Aber davon reden wir nicht, denn wir können z.B. immer ein und die selbe Lichtquelle im Unendlichen und einen statischen Beobachter am Horizont annehmen. Also ist es tatsächlich der gravitative Dopplereffekt (den man übrigens auf der Erde messen kann, soweit ich weiß eine Anwendung des Mössbauer-Effektes).

Wenn ich den Empfänger als statisch am Horizont r=2M sitzend postuliere, dann bedeutet das in einem Raum-Zeit-Diagramm (Kruskal- oder Carter-Penrose-Diagramm), dass er sich tatsächlich mit Lichtgeschwindigkeit bewegt (will heißen, der Horizont ist keine raumartige sondern eine lichtartige Fläche). Klingt widersinnig, ist aber so. Das ist aber nun genau mein Problem: Nach dieser Berechnung müsste dieser Beobachter mein Referenzsystem für alle einfallenden Teilchen sein - das krieg ich aber mathematisch nicht hin, ich kann nur mit einem "im unendlichen ruhenden Beobachter" oder mit einem "frei fallenden Beobachter" rechnen. Im ersten Fall kann ich die Traketorien nicht bis an den Horizont fortsetzen, da die Koordinaten dort singulär werden, in zweiten Fall habe ich kein Referenzsystem mehr, da ja der frei fallende Beobachter (dessen Geschwindigkeit v ich bei r=2M berechnen möchte) selbst das Referenzsystem darstellt - und somit trivialerweise immer v=0 ist.

Den statisch am Horizont sitzenden Beobachter gibt es physikalisch nicht, da er eine unendliche Beschleunigung haben müsste, um an Ort und Stelle zu bleiben, aber mathematisch geht das sicher!

Kommen wir nochmal auf die ursprüngliche Frage und Rays Antwort zurück
Relativisten haben bereits in den 1970er Jahren gezeigt, dass ein Testteilchen den Horizont in radialer Richtung exakt mit c überquert. Ich habe das in einem Paper von 2004 nochmal vorgerechnet:
http://arxiv.org/abs/astro-ph/0309832
(S.8, rechte Spalte)
Ich werde nochmal Ray direkt fragen, denn es geht eigentlich nur um das Verständnis des ZAMOs und der Herleitung von (27) bzw. den einfacheren Fall für Schwarzsschild-Lösung anstelle der komplizierten Kerr-Lösung.
Gruß
Tom

Der Wert eines Dialogs hängt vor allem von der Vielfalt der konkurrierenden Meinungen ab.
Sir Karl R. Popper

Benutzeravatar
Ray Light
hat sich hier eingelebt
hat sich hier eingelebt
Beiträge: 339
Registriert: 25. Sep 2006, 17:13
Wohnort: München

Beitrag von Ray Light » 6. Apr 2008, 12:34

Hallo zusammen. :frosty:

@Skeltek, @Tom
Bitte nicht mit dem Doppler-Effekt durcheinanderkommen. Denn gibt es zwar auch (als Doppler-Blau- oder Doppler-Rotverschiebung) bei allen Strahlungsquellen, die sich um ein Schwarzes Loch bewegen. Die Schwärze des Loches und die Abdunklung von Strahlungsquellen nahe und am Horizont wird jedoch von der Gravitationsrotverschiebung verursacht. Das ist ein Effekt von der Allgemeinen Relativitätstheorie und nicht mit der Newtonschen Gravitationsphysik zu verstehen. Es ist kein kinematischer Effekt wie der Doppler-Effekt.
Tom, zur Nomenklatur: der "gravitative Doppler-Effekt" ist meines Wissens keine Begrifflichkeit der Physik-Literatur. Ich würde aus o.g. Gründen von der Verwendung des Begriffs abraten, weil er den tatsächlichen Sachverhalt verschleiert und Verwirrung stiftet. Es ist eine Gravitationsrotverschiebung oder gravitative Rotverschiebung der ART.

@Tom
Ich wollte Dir nochmals mein 2004 paper (bzw. die pioneering papers von Bardeen und Carter aus den 1970ern) ans Herz lesen. Falls es nicht klar wird, wie man auf Gl. (27) kommt, stehe ich gerne (auch bilateral per E-Mail) zur Verfügung.

Gruß,
Ray
Wir haben verlernt uns zu wundern.

Antworten